LSAT and Law School Admissions Forum

Get expert LSAT preparation and law school admissions advice from PowerScore Test Preparation.

 Administrator
PowerScore Staff
  • PowerScore Staff
  • Posts: 8916
  • Joined: Feb 02, 2011
|
#24312
Complete Question Explanation

Must be True. The correct answer choice is (B)

The economist is basically arguing that no matter the program, the overall level of personal savings does not change – the only thing affected is where that money ends up.

Answer choice (A): There is no evidence in the passage about motives of those backing the tax-incentive proposal.

Answer choice (B): This is the correct answer choice. If no additional money is being invested, money is just changing hands and there will be no way to compensate for the lost tax revenues – this is true no matter how small the actual loss is.

Answer choice (C): The current program only generates small tax revenues losses – we don’t know what would happen in a different program that caused greater loss.

Answer choice (D): This is too sweeping. We only know economic growth would be enhanced by more savings, we don’t have enough evidence to conclude the economy will be in danger otherwise.

Answer choice (E): This is also too sweeping. We only know about the effectiveness of this tax-inventive program. Surely the government has other ways it could encourage saving. For example, they could offer to never tax savings or offer a free car to everyone who saves a certain amount, etc.
 Echx73
  • Posts: 36
  • Joined: Nov 11, 2015
|
#21671
TeamPowerScore,

I am going through the PowerScore LSAT LR Question Type Training Volume 1 book. I am only able to see an explanation for the answer selection, just which answer is correct. I know going through every answer would take way too much of your time, so I will just ask a few.

MBT Page 34 Q#45 The correct answer is B. The proposed tax incentive is unlikely to attract enough additional money into personal savings accounts to make up for the attendant loss in tax revenue. I crossed this off as a possible correct answer because I could prove the the first part of the answer, but I could not prove " to make up for the attendant loss in tax revenue part of the question. Can you fill me in on what I am missing?

Sincerely, I thank you so much for your help!

Eric
 Nikki Siclunov
PowerScore Staff
  • PowerScore Staff
  • Posts: 1362
  • Joined: Aug 02, 2011
|
#21733
Eric,

According to the stimulus, the tax-incentive program (similar to a Traditional IRA, btw) is unlikely to increase how much people save, because they basically shift the money they would have put into their regular Savings accounts into the IRA. People aren't saving more money, they are just taking advantage of the tax break. Meanwhile, this is costing the country tax revenue, because the IRA savings are exempt from income taxation until the money is withdrawn.

If all of this is true, then the cost of the program (loss in tax revenues) probably outweighs the benefits (attract enough additional money into personal savings accounts) - a prephrase that agrees with answer choice (B).

Does that make sense?

Thanks!
 bk1111
  • Posts: 103
  • Joined: Apr 22, 2017
|
#40166
Administrator wrote:
Answer choice (C): The current program only generates small tax revenues losses – we don’t know what would happen in a different program that caused greater loss.
Hi, can someone explain why C is incorrect. Specifically, how do we know the current program discussed in the stimulus only generates a small tax revenues loss?
 Jennifer Janowsky
PowerScore Staff
  • PowerScore Staff
  • Posts: 90
  • Joined: Aug 20, 2017
|
#40185
Hi, bk1111! In the stimulus, it is mentioned here that the current program only generates a small tax revenues loss: "Backers of this proposal claim that its implementation would increase the amount of money available for banks to loan at a relatively small cost to the government in lost tax revenues."

Therefore, it is mentioned that the current program resulted in a small loss, but nothing about the large tax loss mentioned in the answer choice: "(C) A tax-incentive program that resulted in substantial loss of tax revenues would be likely to generate a large increase in personal savings."

Hope this clears it up for you a little bit!
User avatar
 fortunateking
  • Posts: 31
  • Joined: Jan 10, 2022
|
#93463
Dear Powerscore people,
I exclude B because the way it put it seems awkward to meby "the additional money won't make up for govenment's tax revenue loss". Why the people's saving money have anything to do with government's revenue loss to say they are not making up (compensate) ? To me they are totally irrelevant. Am i missing sth?
Thx
 Adam Tyson
PowerScore Staff
  • PowerScore Staff
  • Posts: 5153
  • Joined: Apr 14, 2011
|
#93470
You may be missing that the backers of the proposal claimed that the lost tax revenues to the government would be relatively small compared to the increased savings, fortunateking. The evidence suggests that there may not be any increase in savings, so any lost tax revenues would not be "relatively small" and would not be compensated for by increased savings. The backers are considering the lost tax revenues to be the cost associated with the proposal.

A good prephrase here (which is the answer you come up with yourself before reading any of the answer choices) would be "the backers of the proposal are wrong." Look for an answer choice that matches that idea of the backers being wrong about their claim, and you'll find that answer B is exactly what you're looking for!

Get the most out of your LSAT Prep Plus subscription.

Analyze and track your performance with our Testing and Analytics Package.